Interpretación de los estados de la ecuación de Dirac para electrones en movimiento

Trato de entender una interpretación física de los cuatro componentes del Dirac 4-spinor para un electrón en movimiento (en el caso más simple, una onda plana). Hay una muy buena pregunta y respuesta sobre las interpretaciones ya en SE. Básicamente se muestra que yendo al marco de reposo del electrón (es decir, pags m = ( mi , 0 , 0 , 0 ) ), se encuentran las cuatro soluciones diferentes:

ψ 1 = norte 1 ( 1 0 0 0 ) mi i mi t , ψ 2 = norte 2 ( 0 1 0 0 ) mi i mi t , ψ 3 = norte 3 ( 0 0 1 0 ) mi i mi t  y  ψ 4 = norte 4 ( 0 0 0 1 ) mi i mi t ,

Donde el ψ 1 y ψ 3 tienen helicidad positiva (proyección del espín en la dirección del impulso) mientras que ψ 2 y ψ 4 tienen helicidad negativa.

Además, el factor de fase mi ± i mi t muestra si el estado tiene energía positiva o negativa, por lo tanto, si es una partícula o antipartícula.

Para electrones en movimiento (en la representación de Dirac ), las soluciones obtienen contribuciones adicionales. Por ejemplo

ψ metro o v mi ( X ) = norte 1 ( 1 0 pags z mi + metro pags X + i pags y mi + metro ) Exp ( i pags m X m )
Tiene un tercer y cuarto componente que no desaparecen. Aquí Dominique escribe:

Cuando el impulso NO es igual a cero, estos diferentes estados se mezclan y no puedes hacer una identificación tan simple. Usualmente se dice que el electrón se convierte en una mezcla de un electrón con positrones cuando comienza a moverse.

Sin embargo, el factor de fase dependiente del tiempo mi i pags m X m todavía corresponde a energía positiva para los cuatro componentes, por lo que no puede interpretarse como

ψ metro o v mi ( X ) norte ( norte 1 ψ 1 + pags z mi + metro ψ 3 + pags X + i pags y mi + metro ψ 4 )

(En estas notas de clase se da un argumento similar : el hecho de que los dos últimos componentes no sean cero no significa que contenga soluciones de "energía negativa" ) .

Por lo tanto mi pregunta es:

Para un electrón en movimiento con helicidad + 1 2 (es decir ψ metro o v mi ( X ) ), ¿cuál es la interpretación de los componentes 3 y 4 que no desaparecen (en la representación de Dirac)?

Esta pregunta se vuelve más relevante físicamente cuando uno considera que no hay onda plana. Entonces, el tercer y cuarto componente podrían tener una distribución de intensidad diferente a la del primer componente.

Estoy interesado tanto en las explicaciones como en la literatura que cubre esta pregunta.

¿Está seguro de que las declaraciones en la primera mitad son con respecto a la misma representación de matriz gamma que en la segunda mitad? De todos modos, si elige la representación quiral, Majorana, Dirac o cualquier otra, sus declaraciones en términos de componentes se verán muy diferentes. La forma correcta de hacer esto no es referirse a los componentes del espinor, sino encontrar representaciones QM relativistas de operadores correspondientes a medidas físicas concretas y clasificar los estados de acuerdo con sus estados propios.
@Void Gracias por el comentario. Me parecería extraño que los componentes de Spinor no tengan una interpretación física. En ese caso, ¿por qué uno elegiría esa representación? Sin embargo, si tiene razón y depende de representaciones específicas, puede ser difícil interpretarlas como propiedades físicas generales. ¿Puede señalar la literatura que habla de eso? Si es así, entonces su comentario sería la respuesta perfecta a mi pregunta. (es decir, "Sin interpretación porque depende de la representación").
+1, NiceDrean, publico una pregunta relacionada pero más general, physics.stackexchange.com/questions/359904/… , volveré para responder tu pregunta más tarde.

Respuestas (3)

Para comprobar la proyección de giro en el i -th eje para el tercer y cuarto componente, solo necesita calcular la cantidad

Σ i ψ , Σ i = diagnóstico ( σ i , σ i )
con (por simplicidad) ψ = ( 0 , 0 , 1 , 0 ) y ψ = ( 0 , 0 , 0 , 1 ) correspondientemente

Para verificar la helicidad de estos componentes, solo necesita calcular la cantidad

( Σ pags ) | pags | ψ
de nuevo por ψ = ( 0 , 0 , 1 , 0 ) y ψ = ( 0 , 0 , 0 , 1 ) .

...Por lo tanto, hay dos soluciones SU(2) independientes ϕ R y ϕ L (que están conectados a los espinores de Weyl). ϕ R tiene helicidad positiva y ϕ L tiene helicidad negativa...

Esta oración contiene dos declaraciones incorrectas, relacionadas entre sí.

Primero, en realidad las representaciones irreductibles "elementales" ϕ L / R se definen como los estados propios de la matriz de quiralidad γ 5 . Este último define la forma en que los espinores se transforman en lorentz, y no tiene nada que ver con la helicidad siempre que la masa metro no es cero En el límite de masa cero, la helicidad y la quiralidad coinciden formalmente.

Segundo, ϕ L / R no son espinores de Weyl. El espinor de Weyl es el que satisface una de las ecuaciones

σ m m ψ = 0 , σ ~ m m ψ = 0 ,     dónde      σ m = ( 1 , σ ) ,   σ ~ m = ( 1 , σ )
Se define para describir partículas sin masa con helicidad definida, y no tiene nada que ver con los espinores. ψ L / R dentro del espinor de Dirac mientras metro 0 .

gracias por la respuesta, es bastante util. Solo para estar seguro, ¿podría darme una interpretación explícita para el tercer y cuarto componente? Creo que se deduce de la primera parte de su respuesta, pero no estoy completamente seguro. ¿También tiene una referencia para un artículo o libro que explica esto con un poco más de detalle? ¡Muchas gracias!
@NiceDean: mi respuesta al menos aclara cómo determinar exactamente los valores del giro y la helicidad a los que corresponden estos componentes, por lo que si esto es lo que le interesa (como me parece), entonces la interpretación explícita sigue. En cuanto a la referencia, no estoy seguro, pero esto puede discutirse en QFT de Peskin (en el párrafo llamado "La solución de la ecuación de Dirac libre").
Gracias por la referencia, eso ayuda. Mi pregunta principal es "Para un electrón en movimiento con helicidad +1/2 (es decir, ψ1 está en reposo), ¿cuál es la interpretación del tercer y cuarto componente que no desaparecen?" Veo en su respuesta algunas propiedades útiles de los electrones explicadas, pero no la interpretación del tercer y cuarto componente que no se desvanece. Esta fue la razón para comenzar la recompensa. Estoy seguro de que para usted está claro, para mí, lamentablemente, todavía no está claro. Entonces, ¿podría responder explícitamente esta pregunta o señalar la literatura donde esto se discute explícitamente? ¡muchas gracias!
@NiceDean: la partícula masiva libre se caracteriza por su 4-momento, momento orbital y giro (y, probablemente, por sus cargas grupales internas). Para el espinor de onda plana (que no tiene un momento orbital definido), la única forma de interpretar el tercer y cuarto componente que no se desvanece es calcular la proyección de espín y/o la helicidad correspondientes. Así que no entiendo qué más quieres aclarar.
Desafortunadamente, esto no responde a mi pregunta. Mi pregunta es sobre la interpretación del tercer y cuarto componente distintos de cero del espinor de Dirac. ψ metro o v mi ( X ) en la representación de Dirac. Aclararé la pregunta y reiniciaré la recompensa.

Considere la representación de Dirac de las matrices gamma, ahí tenemos

γ 0 = ( yo 2 0 0 yo 2 )
y las soluciones correspondientes a partículas estáticas y antipartículas pags m = ( ± metro , 0 , 0 , 0 ) son
( α β 0 0 ) mi i metro t , ( 0 0 ϵ d ) mi i metro t
dónde α , β , ϵ , d son algunas constantes. Por supuesto, estos estados propios aún corresponden a dos estados posibles para cada partícula/antipartícula y físicamente esto corresponde a la posibilidad de que la partícula tenga dos signos de espín intrínseco.

Pero primero veamos qué sucede si elegimos otra representación. En la representación quiral , tenemos

γ 0 = ( 0 yo 2 yo 2 0 )
Cuando calculamos los estados estacionarios de partículas/antipartículas, ahora obtenemos
( α β α β ) mi i metro t , ( ϵ d ϵ d ) mi i metro t
dónde α , β , ϵ , d son de nuevo algunas constantes. Entonces puede ver que aunque los componentes "superior" e "inferior" en la representación de Dirac corresponden a partículas frente a antipartículas, la imagen es bastante diferente en la representación quiral de matrices gamma.

Hay una cantidad infinita de representaciones posibles de las matrices gamma relacionadas por transformaciones unitarias y todo lo que discutamos sobre los componentes espinoriales dependerá de la representación elegida.


Para completar el argumento, clasifiquemos los estados propios de energía en la representación de Dirac según su proyección de espín en un cierto eje. El operador de espín intrínseco está dado por Σ i = i γ 0 γ i γ 5 en cualquier representación. En la representación de Dirac (así como en las otras dos representaciones principales quirales y de Majorana), el operador de espín termina como

Σ i = ( σ i 0 0 σ i )
Elegimos ordenar nuestros estados de acuerdo con su componente z, porque lo más conveniente es que tengamos
σ z = ( 1 0 0 1 )
Entonces vemos fácilmente que el componente superior de cada estado propio de energía en la representación de Dirac tiene una proyección positiva de espín en el eje z, mientras que el componente inferior corresponde a una partícula con una proyección negativa de espín en z. Pero repetiré una vez más que en la representación quiral o en cualquier otra interpretación interpretaríamos los componentes de manera bastante diferente.


En cuanto a la respuesta que cita, allí se obtienen las soluciones para las partículas en movimiento al elegir estas estacionarias ± 1 / 2 girar estados y potenciarlos mediante una transformación de Lorentz en soluciones móviles.

En otras palabras, la interpretación de las soluciones

ψ 1 ( X ) = norte 1 ( 1 0 pags z mi + metro pags X + i pags y mi + metro ) Exp ( i pags m X m ) , ψ 2 ( X ) = norte 2 ( 0 1 pags X i pags y mi + metro pags z mi + metro ) Exp ( i pags m X m )
ψ 3 ( X ) = norte 3 ( pags z mi metro pags X + i pags y mi metro 1 0 ) Exp ( i pags m X m ) , ψ 4 ( X ) = norte 4 ( pags X i pags y mi metro pags z mi metro 0 1 ) Exp ( i pags m X m )
es que corresponden a partículas/antipartículas que, en su marco de reposo , tienen una proyección negativa/positiva de espín en el eje z.

Esta es una construcción común para la base de las soluciones generales de la ecuación de Dirac; en otros casos, las personas suelen optar por clasificar las soluciones según su quiralidad.

Gracias por tu respuesta. Es importante lo que escribiste sobre las diferentes representaciones de la ecuación de Dirac. Usé en mi pregunta la representación de Dirac. Luego, en la tercera parte de su pregunta, da una interpretación de los cuatro espinores posibles, pero no responde mi pregunta sobre la interpretación del tercer y cuarto componente de ψ 1 ( X ) . Aclararé la pregunta y reiniciaré la recompensa. Espero que luego pueda responder directamente a mi pregunta sobre la interpretación del tercer y cuarto componente distintos de cero en de ψ 1 ( X ) en la representación de Dirac para p>0.
No me queda claro qué es exactamente lo que estás buscando. Verá que 1) los componentes individuales no tienen un significado de representación invariable, y 2) en la representación de Dirac y el estado de impulso puro y agudo, los "otros" componentes, como el tercer y cuarto componente de ψ 1 ( X ) tienen (hasta la fase y la normalización) el significado físico de pags z / ( mi + metro ) o ( pags X + i pags y ) / ( mi + metro ) . pags X , pags y , pags z , mi , y metro son todos observables físicos duros y le dan un significado directo del componente sin margen de maniobra para ninguna otra interpretación.
Es decir, en un punto dado en el espacio de cantidad de movimiento y en una representación dada, el significado y la interpretación de los componentes se fijan absoluta y positivamente. Pero no existe un significado universal de un solo componente en el espacio de momento y las representaciones.
Como mencioné, me interesa qué significan estos componentes en la representación de Dirac. Por supuesto, tienen algún significado físico allí, pero todavía no lo veo. (Creo que la interpretación debería quedar clara a partir de la derivación de la ecuación de Dirac en esa representación).

La ecuación de Dirac se resuelve utilizando el álgebra de Clifford, en particular las matrices gamma. Hay 16 matrices gamma 4x4 que forman la base requerida. Puede elegir qué base usar (principalmente dependiendo del problema de física que se resuelva) y es posible transformar entre bases.

En la base de Weyl (representación quiral de la ecuación de Dirac) la matriz gamma conocida convencionalmente como γ 5 es diagonal γ 5 = ( yo 0 0 yo ) . Cuando se aplica al espinor componente Dirac 4, en cualquier base lo descompondrá en dos partes, la mano izquierda y la mano derecha. En la base de Weyl, debido a que es diagonal, no mezclará las dos partes del espinor, por lo que podemos decir que los dos componentes superiores del espinor de Dirac representan el campo de la izquierda y los dos inferiores el derecho, es decir.

γ 5 ( ψ R ψ L ) = ( ψ R ψ L )

En la base de Dirac, utilizada en su pregunta, γ 0 es diagonal (y γ 5 no es diagonal).

γ 0 = ( yo 0 0 yo )

γ 0 es el operador de paridad (en todas las bases). Entonces, el espinor de Dirac se divide en dos partes en la base de Dirac, una con paridad par y otra con paridad impar.

Entonces, para responder a la pregunta, los componentes 3 y 4 de ψ metro o v mi representa la parte del espinor con paridad impar en esta solución.

En su pregunta, insinúa que quizás un electrón en movimiento contiene una combinación de estados de energía positivos y negativos. Este no es el caso. Las preguntas y respuestas en zitterbewegung discuten esto especialmente cuando se construyen paquetes de ondas a partir de estados.